Cauchy sequence and subsequence











up vote
2
down vote

favorite












"Let ${x_n}subset U$ be a Cauchy sequence. Give a direct proof that if a subsequence ${x_{nk}}subset S$ has a limit $L$, then the Cauchy sequence ${x_n}subset U$ has $L$ as a limit. Do not assume that general Cauchy sequences are convergent."



Ok I know a lot of people hate helping others with homework when they feel that the person asking the question hasn't worked on the problem. I have spent about 30 minutes going through all my theorems from my textbook on Cauchy sequences and I don't know where to start. Please help me










share|cite|improve this question









New contributor




Sam Cole is a new contributor to this site. Take care in asking for clarification, commenting, and answering.
Check out our Code of Conduct.




















  • Hint: You want to show that the cauchy sequence is close to $L$ from some point onwards. How can you do that using the closeness you get from the subsequence?
    – NL1992
    3 hours ago










  • is that using tails?
    – Sam Cole
    3 hours ago










  • You have that $|x_{n_k}-L|<epsilon$ for all $k>K$ and $|x_n-x_m|<epsilon$ for all $n,m>N$. How can you translate that for $|x_n-L|<epsilon$?
    – NL1992
    3 hours ago















up vote
2
down vote

favorite












"Let ${x_n}subset U$ be a Cauchy sequence. Give a direct proof that if a subsequence ${x_{nk}}subset S$ has a limit $L$, then the Cauchy sequence ${x_n}subset U$ has $L$ as a limit. Do not assume that general Cauchy sequences are convergent."



Ok I know a lot of people hate helping others with homework when they feel that the person asking the question hasn't worked on the problem. I have spent about 30 minutes going through all my theorems from my textbook on Cauchy sequences and I don't know where to start. Please help me










share|cite|improve this question









New contributor




Sam Cole is a new contributor to this site. Take care in asking for clarification, commenting, and answering.
Check out our Code of Conduct.




















  • Hint: You want to show that the cauchy sequence is close to $L$ from some point onwards. How can you do that using the closeness you get from the subsequence?
    – NL1992
    3 hours ago










  • is that using tails?
    – Sam Cole
    3 hours ago










  • You have that $|x_{n_k}-L|<epsilon$ for all $k>K$ and $|x_n-x_m|<epsilon$ for all $n,m>N$. How can you translate that for $|x_n-L|<epsilon$?
    – NL1992
    3 hours ago













up vote
2
down vote

favorite









up vote
2
down vote

favorite











"Let ${x_n}subset U$ be a Cauchy sequence. Give a direct proof that if a subsequence ${x_{nk}}subset S$ has a limit $L$, then the Cauchy sequence ${x_n}subset U$ has $L$ as a limit. Do not assume that general Cauchy sequences are convergent."



Ok I know a lot of people hate helping others with homework when they feel that the person asking the question hasn't worked on the problem. I have spent about 30 minutes going through all my theorems from my textbook on Cauchy sequences and I don't know where to start. Please help me










share|cite|improve this question









New contributor




Sam Cole is a new contributor to this site. Take care in asking for clarification, commenting, and answering.
Check out our Code of Conduct.











"Let ${x_n}subset U$ be a Cauchy sequence. Give a direct proof that if a subsequence ${x_{nk}}subset S$ has a limit $L$, then the Cauchy sequence ${x_n}subset U$ has $L$ as a limit. Do not assume that general Cauchy sequences are convergent."



Ok I know a lot of people hate helping others with homework when they feel that the person asking the question hasn't worked on the problem. I have spent about 30 minutes going through all my theorems from my textbook on Cauchy sequences and I don't know where to start. Please help me







proof-writing cauchy-sequences






share|cite|improve this question









New contributor




Sam Cole is a new contributor to this site. Take care in asking for clarification, commenting, and answering.
Check out our Code of Conduct.











share|cite|improve this question









New contributor




Sam Cole is a new contributor to this site. Take care in asking for clarification, commenting, and answering.
Check out our Code of Conduct.









share|cite|improve this question




share|cite|improve this question








edited 3 hours ago









Boshu

619315




619315






New contributor




Sam Cole is a new contributor to this site. Take care in asking for clarification, commenting, and answering.
Check out our Code of Conduct.









asked 3 hours ago









Sam Cole

162




162




New contributor




Sam Cole is a new contributor to this site. Take care in asking for clarification, commenting, and answering.
Check out our Code of Conduct.





New contributor





Sam Cole is a new contributor to this site. Take care in asking for clarification, commenting, and answering.
Check out our Code of Conduct.






Sam Cole is a new contributor to this site. Take care in asking for clarification, commenting, and answering.
Check out our Code of Conduct.












  • Hint: You want to show that the cauchy sequence is close to $L$ from some point onwards. How can you do that using the closeness you get from the subsequence?
    – NL1992
    3 hours ago










  • is that using tails?
    – Sam Cole
    3 hours ago










  • You have that $|x_{n_k}-L|<epsilon$ for all $k>K$ and $|x_n-x_m|<epsilon$ for all $n,m>N$. How can you translate that for $|x_n-L|<epsilon$?
    – NL1992
    3 hours ago


















  • Hint: You want to show that the cauchy sequence is close to $L$ from some point onwards. How can you do that using the closeness you get from the subsequence?
    – NL1992
    3 hours ago










  • is that using tails?
    – Sam Cole
    3 hours ago










  • You have that $|x_{n_k}-L|<epsilon$ for all $k>K$ and $|x_n-x_m|<epsilon$ for all $n,m>N$. How can you translate that for $|x_n-L|<epsilon$?
    – NL1992
    3 hours ago
















Hint: You want to show that the cauchy sequence is close to $L$ from some point onwards. How can you do that using the closeness you get from the subsequence?
– NL1992
3 hours ago




Hint: You want to show that the cauchy sequence is close to $L$ from some point onwards. How can you do that using the closeness you get from the subsequence?
– NL1992
3 hours ago












is that using tails?
– Sam Cole
3 hours ago




is that using tails?
– Sam Cole
3 hours ago












You have that $|x_{n_k}-L|<epsilon$ for all $k>K$ and $|x_n-x_m|<epsilon$ for all $n,m>N$. How can you translate that for $|x_n-L|<epsilon$?
– NL1992
3 hours ago




You have that $|x_{n_k}-L|<epsilon$ for all $k>K$ and $|x_n-x_m|<epsilon$ for all $n,m>N$. How can you translate that for $|x_n-L|<epsilon$?
– NL1992
3 hours ago










3 Answers
3






active

oldest

votes

















up vote
2
down vote













Let $x_n$ be a Cauchy sequence and $(x_n)_k$ a subsequence. Let $epsilon >0$. Then for sufficiently large $n$, since $x_n$ is Cauchy, $|x_n-(x_n)_k|<epsilon /2.$ Since the subsequence converges to $L$ we have $|(x_n)_k-L|<epsilon /2$ for sufficiently large $k$. Choosing the maximum of $n$ for which the above are satisfied gives $|x_n-L|=|x_n-(x_n)_k+(x_n)_k-L|leq |x_n-(x_n)_k|+|(x_n)_k-L|<epsilon$.






share|cite|improve this answer




























    up vote
    1
    down vote













    The actual theorem states that a Cauchy sequence is convergent iff it has a convergent subsequence, and your question only asks us to prove the if part. It follows rather immediately from the triangle inequality. Pick $n_k$ such that $|x_{n_k}-L|leqdfrac{epsilon}{2}$, and $n,m$ such that $|x_n-x_m|leq dfrac{epsilon}{2}$. You should be able to see why these exist for integers sufficiently large. Then



    $$|x_n-L|leq|x_n-x_m|+|x_{n_k}-L|$$.






    share|cite|improve this answer




























      up vote
      1
      down vote













      Fix $epsilon>0.$ $x_{n_k}rightarrow L$ implies there is an integer $k_0$ such that $d(x_{n_k},L)<epsilon/2$ for $kgeq k_0.$ $(x_n)$ is Cauchy implies there is an integer $n_0$ such that $d(x_m,x_n)<epsilon/2$ for $m,ngeq n_0.$ Take $N=max{n_0,n_{k_0}}.$ Then for $ngeq N,$ $d(x_n,L)leq d(x_n,x_{n_{k_0}})+d(x_{n_{k_0}},L)<epsilon.$ Hence $x_nrightarrow L.$






      share|cite|improve this answer





















        Your Answer





        StackExchange.ifUsing("editor", function () {
        return StackExchange.using("mathjaxEditing", function () {
        StackExchange.MarkdownEditor.creationCallbacks.add(function (editor, postfix) {
        StackExchange.mathjaxEditing.prepareWmdForMathJax(editor, postfix, [["$", "$"], ["\\(","\\)"]]);
        });
        });
        }, "mathjax-editing");

        StackExchange.ready(function() {
        var channelOptions = {
        tags: "".split(" "),
        id: "69"
        };
        initTagRenderer("".split(" "), "".split(" "), channelOptions);

        StackExchange.using("externalEditor", function() {
        // Have to fire editor after snippets, if snippets enabled
        if (StackExchange.settings.snippets.snippetsEnabled) {
        StackExchange.using("snippets", function() {
        createEditor();
        });
        }
        else {
        createEditor();
        }
        });

        function createEditor() {
        StackExchange.prepareEditor({
        heartbeatType: 'answer',
        convertImagesToLinks: true,
        noModals: true,
        showLowRepImageUploadWarning: true,
        reputationToPostImages: 10,
        bindNavPrevention: true,
        postfix: "",
        imageUploader: {
        brandingHtml: "Powered by u003ca class="icon-imgur-white" href="https://imgur.com/"u003eu003c/au003e",
        contentPolicyHtml: "User contributions licensed under u003ca href="https://creativecommons.org/licenses/by-sa/3.0/"u003ecc by-sa 3.0 with attribution requiredu003c/au003e u003ca href="https://stackoverflow.com/legal/content-policy"u003e(content policy)u003c/au003e",
        allowUrls: true
        },
        noCode: true, onDemand: true,
        discardSelector: ".discard-answer"
        ,immediatelyShowMarkdownHelp:true
        });


        }
        });






        Sam Cole is a new contributor. Be nice, and check out our Code of Conduct.










        draft saved

        draft discarded


















        StackExchange.ready(
        function () {
        StackExchange.openid.initPostLogin('.new-post-login', 'https%3a%2f%2fmath.stackexchange.com%2fquestions%2f3023499%2fcauchy-sequence-and-subsequence%23new-answer', 'question_page');
        }
        );

        Post as a guest















        Required, but never shown

























        3 Answers
        3






        active

        oldest

        votes








        3 Answers
        3






        active

        oldest

        votes









        active

        oldest

        votes






        active

        oldest

        votes








        up vote
        2
        down vote













        Let $x_n$ be a Cauchy sequence and $(x_n)_k$ a subsequence. Let $epsilon >0$. Then for sufficiently large $n$, since $x_n$ is Cauchy, $|x_n-(x_n)_k|<epsilon /2.$ Since the subsequence converges to $L$ we have $|(x_n)_k-L|<epsilon /2$ for sufficiently large $k$. Choosing the maximum of $n$ for which the above are satisfied gives $|x_n-L|=|x_n-(x_n)_k+(x_n)_k-L|leq |x_n-(x_n)_k|+|(x_n)_k-L|<epsilon$.






        share|cite|improve this answer

























          up vote
          2
          down vote













          Let $x_n$ be a Cauchy sequence and $(x_n)_k$ a subsequence. Let $epsilon >0$. Then for sufficiently large $n$, since $x_n$ is Cauchy, $|x_n-(x_n)_k|<epsilon /2.$ Since the subsequence converges to $L$ we have $|(x_n)_k-L|<epsilon /2$ for sufficiently large $k$. Choosing the maximum of $n$ for which the above are satisfied gives $|x_n-L|=|x_n-(x_n)_k+(x_n)_k-L|leq |x_n-(x_n)_k|+|(x_n)_k-L|<epsilon$.






          share|cite|improve this answer























            up vote
            2
            down vote










            up vote
            2
            down vote









            Let $x_n$ be a Cauchy sequence and $(x_n)_k$ a subsequence. Let $epsilon >0$. Then for sufficiently large $n$, since $x_n$ is Cauchy, $|x_n-(x_n)_k|<epsilon /2.$ Since the subsequence converges to $L$ we have $|(x_n)_k-L|<epsilon /2$ for sufficiently large $k$. Choosing the maximum of $n$ for which the above are satisfied gives $|x_n-L|=|x_n-(x_n)_k+(x_n)_k-L|leq |x_n-(x_n)_k|+|(x_n)_k-L|<epsilon$.






            share|cite|improve this answer












            Let $x_n$ be a Cauchy sequence and $(x_n)_k$ a subsequence. Let $epsilon >0$. Then for sufficiently large $n$, since $x_n$ is Cauchy, $|x_n-(x_n)_k|<epsilon /2.$ Since the subsequence converges to $L$ we have $|(x_n)_k-L|<epsilon /2$ for sufficiently large $k$. Choosing the maximum of $n$ for which the above are satisfied gives $|x_n-L|=|x_n-(x_n)_k+(x_n)_k-L|leq |x_n-(x_n)_k|+|(x_n)_k-L|<epsilon$.







            share|cite|improve this answer












            share|cite|improve this answer



            share|cite|improve this answer










            answered 3 hours ago









            AnyAD

            1,831811




            1,831811






















                up vote
                1
                down vote













                The actual theorem states that a Cauchy sequence is convergent iff it has a convergent subsequence, and your question only asks us to prove the if part. It follows rather immediately from the triangle inequality. Pick $n_k$ such that $|x_{n_k}-L|leqdfrac{epsilon}{2}$, and $n,m$ such that $|x_n-x_m|leq dfrac{epsilon}{2}$. You should be able to see why these exist for integers sufficiently large. Then



                $$|x_n-L|leq|x_n-x_m|+|x_{n_k}-L|$$.






                share|cite|improve this answer

























                  up vote
                  1
                  down vote













                  The actual theorem states that a Cauchy sequence is convergent iff it has a convergent subsequence, and your question only asks us to prove the if part. It follows rather immediately from the triangle inequality. Pick $n_k$ such that $|x_{n_k}-L|leqdfrac{epsilon}{2}$, and $n,m$ such that $|x_n-x_m|leq dfrac{epsilon}{2}$. You should be able to see why these exist for integers sufficiently large. Then



                  $$|x_n-L|leq|x_n-x_m|+|x_{n_k}-L|$$.






                  share|cite|improve this answer























                    up vote
                    1
                    down vote










                    up vote
                    1
                    down vote









                    The actual theorem states that a Cauchy sequence is convergent iff it has a convergent subsequence, and your question only asks us to prove the if part. It follows rather immediately from the triangle inequality. Pick $n_k$ such that $|x_{n_k}-L|leqdfrac{epsilon}{2}$, and $n,m$ such that $|x_n-x_m|leq dfrac{epsilon}{2}$. You should be able to see why these exist for integers sufficiently large. Then



                    $$|x_n-L|leq|x_n-x_m|+|x_{n_k}-L|$$.






                    share|cite|improve this answer












                    The actual theorem states that a Cauchy sequence is convergent iff it has a convergent subsequence, and your question only asks us to prove the if part. It follows rather immediately from the triangle inequality. Pick $n_k$ such that $|x_{n_k}-L|leqdfrac{epsilon}{2}$, and $n,m$ such that $|x_n-x_m|leq dfrac{epsilon}{2}$. You should be able to see why these exist for integers sufficiently large. Then



                    $$|x_n-L|leq|x_n-x_m|+|x_{n_k}-L|$$.







                    share|cite|improve this answer












                    share|cite|improve this answer



                    share|cite|improve this answer










                    answered 3 hours ago









                    Boshu

                    619315




                    619315






















                        up vote
                        1
                        down vote













                        Fix $epsilon>0.$ $x_{n_k}rightarrow L$ implies there is an integer $k_0$ such that $d(x_{n_k},L)<epsilon/2$ for $kgeq k_0.$ $(x_n)$ is Cauchy implies there is an integer $n_0$ such that $d(x_m,x_n)<epsilon/2$ for $m,ngeq n_0.$ Take $N=max{n_0,n_{k_0}}.$ Then for $ngeq N,$ $d(x_n,L)leq d(x_n,x_{n_{k_0}})+d(x_{n_{k_0}},L)<epsilon.$ Hence $x_nrightarrow L.$






                        share|cite|improve this answer

























                          up vote
                          1
                          down vote













                          Fix $epsilon>0.$ $x_{n_k}rightarrow L$ implies there is an integer $k_0$ such that $d(x_{n_k},L)<epsilon/2$ for $kgeq k_0.$ $(x_n)$ is Cauchy implies there is an integer $n_0$ such that $d(x_m,x_n)<epsilon/2$ for $m,ngeq n_0.$ Take $N=max{n_0,n_{k_0}}.$ Then for $ngeq N,$ $d(x_n,L)leq d(x_n,x_{n_{k_0}})+d(x_{n_{k_0}},L)<epsilon.$ Hence $x_nrightarrow L.$






                          share|cite|improve this answer























                            up vote
                            1
                            down vote










                            up vote
                            1
                            down vote









                            Fix $epsilon>0.$ $x_{n_k}rightarrow L$ implies there is an integer $k_0$ such that $d(x_{n_k},L)<epsilon/2$ for $kgeq k_0.$ $(x_n)$ is Cauchy implies there is an integer $n_0$ such that $d(x_m,x_n)<epsilon/2$ for $m,ngeq n_0.$ Take $N=max{n_0,n_{k_0}}.$ Then for $ngeq N,$ $d(x_n,L)leq d(x_n,x_{n_{k_0}})+d(x_{n_{k_0}},L)<epsilon.$ Hence $x_nrightarrow L.$






                            share|cite|improve this answer












                            Fix $epsilon>0.$ $x_{n_k}rightarrow L$ implies there is an integer $k_0$ such that $d(x_{n_k},L)<epsilon/2$ for $kgeq k_0.$ $(x_n)$ is Cauchy implies there is an integer $n_0$ such that $d(x_m,x_n)<epsilon/2$ for $m,ngeq n_0.$ Take $N=max{n_0,n_{k_0}}.$ Then for $ngeq N,$ $d(x_n,L)leq d(x_n,x_{n_{k_0}})+d(x_{n_{k_0}},L)<epsilon.$ Hence $x_nrightarrow L.$







                            share|cite|improve this answer












                            share|cite|improve this answer



                            share|cite|improve this answer










                            answered 3 hours ago









                            John_Wick

                            1,104111




                            1,104111






















                                Sam Cole is a new contributor. Be nice, and check out our Code of Conduct.










                                draft saved

                                draft discarded


















                                Sam Cole is a new contributor. Be nice, and check out our Code of Conduct.













                                Sam Cole is a new contributor. Be nice, and check out our Code of Conduct.












                                Sam Cole is a new contributor. Be nice, and check out our Code of Conduct.
















                                Thanks for contributing an answer to Mathematics Stack Exchange!


                                • Please be sure to answer the question. Provide details and share your research!

                                But avoid



                                • Asking for help, clarification, or responding to other answers.

                                • Making statements based on opinion; back them up with references or personal experience.


                                Use MathJax to format equations. MathJax reference.


                                To learn more, see our tips on writing great answers.





                                Some of your past answers have not been well-received, and you're in danger of being blocked from answering.


                                Please pay close attention to the following guidance:


                                • Please be sure to answer the question. Provide details and share your research!

                                But avoid



                                • Asking for help, clarification, or responding to other answers.

                                • Making statements based on opinion; back them up with references or personal experience.


                                To learn more, see our tips on writing great answers.




                                draft saved


                                draft discarded














                                StackExchange.ready(
                                function () {
                                StackExchange.openid.initPostLogin('.new-post-login', 'https%3a%2f%2fmath.stackexchange.com%2fquestions%2f3023499%2fcauchy-sequence-and-subsequence%23new-answer', 'question_page');
                                }
                                );

                                Post as a guest















                                Required, but never shown





















































                                Required, but never shown














                                Required, but never shown












                                Required, but never shown







                                Required, but never shown

































                                Required, but never shown














                                Required, but never shown












                                Required, but never shown







                                Required, but never shown







                                Popular posts from this blog

                                サソリ

                                広島県道265号伴広島線

                                Setup Asymptote in Texstudio